Mathcenter Forum

Mathcenter Forum (https://www.mathcenter.net/forum/index.php)
-   พีชคณิต (https://www.mathcenter.net/forum/forumdisplay.php?f=15)
-   -   Trigonometric Marathon (https://www.mathcenter.net/forum/showthread.php?t=1286)

Mastermander 16 เมษายน 2006 20:57

Trigonometric Marathon
 
เห็นมีกระแสโจทย์ตรีโกณมา ก็เลยมาตั้งกระทู้นี้เพื่อทำโจทย์ต่อเนื่องครับ
เริ่มเลยครับ

1. กำหนด $\; \sec\theta\csc\theta=K $ จงหาค่าของ$$\tan^5\theta+\cot^5\theta$$

M@gpie 16 เมษายน 2006 22:00

เนื่องจาก \( \tan \theta + \frac{1}{\tan \theta} = \frac{\tan ^2 \theta +1 }{\tan \theta} = \sec \theta \csc \theta = K \; \)และ \(\; \tan^5 \theta + \frac{1}{\tan^5 \theta} = \tan^5 \theta + \cot^5 \theta \; \)
เพื่อความสะดวกจะเปลี่ยนตัวแปรให้โจทย์เป็น \(\; x+\frac{1}{x} = K \) จงหาค่าของ \( x^5 +\frac{1}{x^5} \; \)
จะได้ว่า \( \; x^2+\frac{1}{x^2} = K^2-2 \) และ \( x^4+\frac{1}{x^4} = (K^2-2)^2 -2 \; \)
เนื่องจาก \( x^5 +\frac{1}{x^5} = (x+\frac{1}{x})(x^4 - x^2 + 1 - \frac{1}{x^2} + \frac{1}{x^4}) = K( (K^2-2)^2 - (K^2 -2) -1 ) \; \; \)
เป็นอันเรียบร้อยครับ ถูกไหม!! อิอิ

ปล: มีการแก้ไขเนื่องจากคิดผิด อิอิ

ข้อต่อปาย
ถ้า \( x+\frac{1}{x} = 2\cos \theta \)
จงแสดงว่า
1.\( x = e^{j \theta} , e^{-j\theta} \)
2.\( x^n+\frac{1}{x^n} = 2 \cos n \theta \)

Mastermander 16 เมษายน 2006 22:59

เนื่องจาก $ e^{j\theta}=\cos\theta+ j \sin\theta $
ดังนั้น $ e^{-j\theta} = \cos\theta - j \sin\theta $
$ e^{j\theta}+e^{-j\theta}=2\cos\theta $

จากทฤษฎีบทของเดอมัวร์ :p
$ (e^{i\theta})^n=\cos n\theta+i\sin n\theta $

จะได้ $ x^n =e^{jn\theta}=\cos n\theta + j\sin n\theta $
และ $ x^{-n} =e^{-jn\theta}=\cos n\theta - j\sin n\theta $

ดังนั้น $ x^n + x^{-n} = 2\cos n\theta $

ข้อ 3.จงพิสูจน์ว่า
$$ \cos^2(\frac{\pi}{8}-A) - \cos^2(\frac{\pi}{8}+A) =\frac{\sin 2A}{\sqrt2} $$


ปล.ของพี่ M@gpie ยังไม่ถูกต้องสมบูรณ์ครับ

Mastermander 17 เมษายน 2006 01:21

4. จงหาจำนวนจริง x ที่สอดคล้องกับ
$$ \frac{\sin^2x-1/4}{\sqrt3-(\sin x+\cos x)}>0 $$

R-Tummykung de Lamar 17 เมษายน 2006 01:40

จากค่าสูงสุดของ $\sin x + \cos x$ คือ $\sqrt 2$
พิสูจน์ ให้ค่าสูงสุดคือ $y$
$$\sin x + \cos x\leq y$$
คูณตลอดด้วย $\sin 45^\circ =\cos 45 ^\circ =\frac{\sqrt{2}}2 >0$
$$\sin x \cos45^\circ + \cos x \sin 45^\circ \leq \frac{\sqrt 2}{2}y$$
$$\sin(x+45^\circ ) \leq \frac{\sqrt 2}{2}y$$
ซึ่งค่าสูงสุดของ $\sin$ คือ $1$ นั่นคือ $\frac{\sqrt 2}{2}y=1$
$\therefore y\ =\ \sqrt 2$
เกิดเมื่อ $x\ =\ 45^\circ + 360n^\circ ,n\in I$


กลับมาที่โจทย์จะได้ว่าที่ส่วนเป็นบวกเสมอ นั่นคือ $(\sin x-\frac12)(\sin x+\frac12)>0$
จะได้ $\sin x \in [-1,-\frac 12)\cup(\frac 12,1]$
นั่นคือ $x \in(30^\circ +180n^\circ ,150^\circ +180n^\circ ) ,n\in I$

nongtum 17 เมษายน 2006 05:12

อ้างอิง:

ข้อความเดิมของคุณ Mastermander:
ข้อ 3.จงพิสูจน์ว่า
$$ \cos^2(\frac{\pi}{8}-A) - \cos^2(\frac{\pi}{8}+A) =\frac{\sin 2A}{\sqrt2} $$

โดยใช้ $\cos^2A=(\cos 2A+1)/2$ จะได้เทอมทางซ้ายมือเป็น
$$\frac{1}{2}[1+\cos(\frac{\pi}{4}-2A)-1-\cos(\frac{\pi}{4}+2A)]
=\frac{1}{2}(2\sin\frac{\pi}{4}\sin 2A)=\frac{1}{\sqrt{2}}\sin 2A$$

5. จงแก้สมการ $$\cos x-\cos 2x+\cos 3x-\cos 4x=\frac{1}2$$

Donovan 17 เมษายน 2006 17:23

ข้อ 5 แปลงได้เป็นสมการกำลัง 4 ครับ แฮ่ๆ :D ( มีสูตรในการแก้อยู่นี่นา )

คือ y^{ 4 } - (1/2)y^{ 3 } - (3/4)y^{ 2 } + (1/4)y + 1/16 = 0

ให้ y = cos x

x = cos^{ -1 } y

แก้สมการกำลัง 4 นี้ได้ ก็จะได้คำตอบครับ

ใครมีวิธีลัดบ้างครับ ช่วยบอกที

nongtum 17 เมษายน 2006 18:08

สมการถูกแล้ว ข้อห้าไม่ต้องใช้สูตรครับ ลองแยกตัวประกอบแล้วสังเกตดีๆ หรือจะคูณสมการโจทย์ด้วยตัวที่เหมาะสมก่อนก็ได้ครับ(ใบ้เยอะแล้ว) ใครเข้ามาอ่านมาตอบโจทย์ที่นี่เป็นประจำคงพอจะนึกออกว่ามันคือ...

Mastermander 17 เมษายน 2006 22:04

$$y^{ 4 } - (1/2)y^{ 3 } - (3/4)y^{ 2 } + (1/4)y + 1/16 = 0$$

เนืี่องจากผลบวกของรากทั้ง 4 เป็น 1/2 จึงได้ข้อสังเกตข้างต้นจากสูตรที่ว่า
$$\sum_{k=1}^n \cos\frac{(2k-1)\pi}{2n+1}=\frac12 $$

และจากการตรวจสอบผลคูณของคำตอบก็พบว่า เท่ากับ 1/16

ดังนั้นจึงสรุปว่ารากของสมการนี้คือ

$$\cos x = \cos \frac{\pi}{9}, \cos \frac{3\pi}{9}, \cos \frac{5\pi}{9}, \cos \frac{7\pi}{9}$$

Mastermander 17 เมษายน 2006 23:59

ขอเฉลยข้อแรกนะครับ

$$\tan \theta + \cot \theta = k = x+\frac{1}{x}$$
$$x^5+\frac{1}{x^5}=(x+\frac{1}{x})(x^4+\frac{1}{x^4})-(x^3+\frac{1}{x^3})$$

$$Answer=K(K^4-4K^2+2)-(K^3-3K)=K^5-5K^3+5K$$

ส่วนคำตอบของข้อ 5 คือ (Edited)
$x=2n\pi\pm\frac{\pi}{3}\;,2n\pi\pm\frac{\pi}{9}\;,2n\pi\pm\frac{5\pi}{9}\;,2n\pi\pm\frac{7\pi}{9} $

6. ถ้า (Edited) $$\sin(a+b)\sin(c-d)=\sin(a-b)\sin(c+d)$$
จงพิสูจน์ว่า $$\cot a \cot d = \cot b \cot c$$
:yum:

nongtum 18 เมษายน 2006 04:27

คิดว่าโจทย์ข้อหกน่าจะผิด เพราะคิดย้อนกลับได้ดังนี้
$$\begin{eqnarray}
\cot a\cot d&=&\cot b\cot c\\
(\cos a\sin b)(\sin c\cos d)&=&(\sin a\cos b)(\cos c\sin d)\\
(\sin (a+b)-\sin (a-b))(\sin (c+d)+\sin (c-d))&=&(\sin (a+b)+\sin (a-b))(\sin (c+d)-\sin (c-d))\\
\end{eqnarray}$$
คูณกระจายเทอมแต่ละข้าง แล้วกำจัดเทอมที่เท่ากันทิ้ง ในที่สุดจะได้ $\sin (a+b)\sin (c-d)=\sin (a-b)\sin (c+d)$

ส่วนข้อห้า คำตอบทั่วไปหายไปอื้อเลยครับ และหากไม่ยุ่งยากเกินไปช่วยพิสูจน์สูตรที่อ้างมาในข้อ 5 ได้ไหมครับ คิดว่าหลายคนน่าจะตามไม่ทัน

Mastermander 18 เมษายน 2006 13:48

โจทย์ข้อ 6 พิมพ์ผิดครับ ตามที่คุณ nongtum ทำไว้แล้ว
\[
\cos \frac{\pi }{{2n + 1}}+\cos \frac{{3\pi }}{{2n + 1}} + \ldots + \cos \frac{{\left( {2n - 1} \right)\pi }}{{2n + 1}} = \varepsilon
\]\[
\left( {2\sin \frac{\pi }{{2n + 1}}} \right)\left( {\cos \frac{\pi }{{2n + 1}}+\cos \frac{{3\pi }}{{2n + 1}} + \ldots + \cos \frac{{\left( {2n - 1} \right)\pi }}{{2n + 1}}} \right) = \left( {2\sin \frac{\pi }{{2n + 1}}} \right)\varepsilon
\]\[
= 2\cos \frac{\pi }{{2n + 1}}\sin \frac{\pi }{{2n + 1}} + 2\cos \frac{{3\pi }}{{2n + 1}}\sin \frac{\pi }{{2n + 1}} + \ldots + 2\cos \frac{{\left( {2n - 1} \right)\pi }}{{2n + 1}}\sin \frac{\pi }{{2n + 1}}
\]\[\because 2\cos \alpha \sin \beta = \sin \left( {\alpha + \beta } \right) - \sin \left( {\alpha - \beta } \right)
\]\[
2\varepsilon \sin \frac{\pi }{{2n + 1}} = \sin \frac{{2\pi }}{{2n + 1}} + \sin \frac{{4\pi }}{{2n + 1}} - \sin \frac{{2n\pi }}{{2n + 1}} + \ldots + \sin \frac{{2n\pi }}{{2n + 1}} - \sin \frac{{\left( {2n - 2} \right)\pi }}{{2n + 1}}
\]\[
2\varepsilon \sin \frac{\pi }{{2n + 1}} = \sin \frac{{2n\pi }}{{2n + 1}}
\]\[\because \sin \left( {\pi - \theta } \right) = \sin \theta \quad \to \quad\sin \left( {\pi - \frac{{2n\pi }}{{2n + 1}}} \right) = \sin \frac{\pi }{{2n + 1}}
\]\[
1 = 2\varepsilon
\]\[
\varepsilon = \sum\limits_{k = 1}^n {\cos \frac{{\left( {2k - 1} \right)\pi }}{{2n + 1}}} = \frac{1}{2}
\]

nongtum 18 เมษายน 2006 14:29

7. จงแก้สมการ $$2^{-\sin^2x}+2^{-\cos^2x}=\sin y+\cos y$$

Mastermander 18 เมษายน 2006 15:00

7.
$$ 2^{\sin^2 x} + 2^{\cos^2 x} = 2\sqrt2 \sin(y+\pi/4) $$
เนื่องจากค่าสูงสุดของ $\sin (y+\pi/4) =1$
และค่าต่ำสุดของ $ 2^{\sin^2 x} + 2^{\cos^2 x} =2\sqrt2 $ ก็ต่อเมื่อ $ \sin^2 x = \cos^2x=1/2 $
และจากโจทย์ซึ่งเป็นจุดสัมพัทธ์ของทั้ง 2 กรณี
จึงทำให้ได้ว่า $ \sin(y+\pi/4)=1 $ และ $\sin^2 x = \cos^2 x = 1/2$

ถูกทางรึเปล่าครับ

nongtum 18 เมษายน 2006 15:20

ถูกแล้วครับ เหลือแต่สรุปคำตอบให้ถูก อ้อ อย่าลืมแก้คำตอบทั่วไปของข้อ 5 ด้วยครับ อุตส่าห์คิดถูกแล้ว อย่าให้ตายตอนจบง่ายๆแบบนั้น

Mastermander 18 เมษายน 2006 19:52

$\sin(y+\pi/4)=1$
$y+\pi/4=\pi/2 +2n\pi$
$y=\pi/2+2n\pi$
$\sin^2x=1/2$
$x=n\pi \pm \pi/4$

8. ในสามเหลี่ยมใดๆ ถ้า
\( \cot A+\sin A \csc B \csc C=m\)
$\cot B+\sin B \csc C \csc A =n$
$\cot C+\sin C \csc A \csc B =p$
จงหาความสัมพันธ์ระหว่าง m , n , p

Mastermander 19 เมษายน 2006 21:03

9. จงหาจำนวนจริง $A$ ที่ทำให้

$\sin A\cos A-\dfrac{\cos A}{\sqrt2} +\dfrac{\sin A}2 -\frac1{2\sqrt2} \le 0$

Mastermander 20 เมษายน 2006 22:35

เฉลยข้อ 8 ซึ่งเฉลยโดยคุณ GFK แห่งวิชาการ.คอม


Mastermander 21 เมษายน 2006 01:26

10.

tunococ 22 เมษายน 2006 13:31

\(7 + 4\sqrt{3}\) ถ้าคิดไม่ผิดนะ (สงสัยจะผิด)

Mastermander 22 เมษายน 2006 14:00

อ้างอิง:

ข้อความเดิมของคุณ tunococ:
\(7 + 4\sqrt{3}\) ถ้าคิดไม่ผิดนะ (สงสัยจะผิด)
ตอบข้อไหนครับ...

(แต่ก็ไม่มีคำตอบข้อไหนเหมือนที่ตอบมาด้วย)

Mastermander 22 เมษายน 2006 20:49

ไม่มีคนทำเลย:sweat:

11.จงหาผลสำเร็จของ

gools 23 เมษายน 2006 13:14

ข้อ 10 ตอบ 1 ครับ :D

Mastermander 23 เมษายน 2006 13:31

อ้างอิง:

ข้อความเดิมของคุณ gools:
ข้อ 10 ตอบ 1 ครับ :D

ยังไม่ถูกนะครับ :sung:

gools 23 เมษายน 2006 13:59

ข้อ 10 ตอบ 3 ครับ :please:
ข้อ 9 ตอบ $\mathbf{R}-[(\frac{\pi}{4}+2n\pi,\frac{2\pi }{3}+2n\pi )\cup (\frac{3\pi }{4}+2n\pi,\frac{4\pi }{3}+2n\pi )]$ เมื่อ $n \in \mathbf{Z}$

Mastermander 23 เมษายน 2006 14:12

อ้างอิง:

ข้อความเดิมของคุณ gools:
ข้อ 10 ตอบ 3 ครับ :please:
ข้อ 9 ตอบ $\mathbf{R}-[(\frac{\pi}{4}+2n\pi,\frac{2\pi }{3}+2n\pi )\cup (\frac{3\pi }{4}+2n\pi,\frac{4\pi }{3}+2n\pi )]$ เมื่อ $n \in \mathbf{Z}$

ข้อ 10 ถูกต้องครับ

ข้อ 9 ถือว่าถูกต้องครับ (ถ้าจะให้สมบูรณ์ต้องตอบเป็นช่วงปิด)

Mastermander 24 เมษายน 2006 14:07

12. จงหาค่าของ
$$\sec^2\frac{\pi}{9}+\sec^2\frac{5\pi}{9}+\sec^2\frac{7\pi}{9}$$

ZiLnIcE 25 เมษายน 2006 01:22

12.ตอบ 36(ถูกหรือป่าวครับ) :confused:

Mastermander 25 เมษายน 2006 08:11

ตอบ 36 เป็นคำตอบที่ถูกต้องครับ

Mastermander 25 เมษายน 2006 15:38

gg

ZiLnIcE 25 เมษายน 2006 15:50

13. กำหนด $$\arctan x-\frac{\pi}{4}=\sum_{k=1}^{2546}\arctan \frac{1}{k^2+k+1}$$ จงหาค่าxที่สอดคล้องกับสมการ

Mastermander 25 เมษายน 2006 22:48

ผมขอเฉลยข้อเก่าละกันครับ

R-Tummykung de Lamar 25 เมษายน 2006 22:55

ข้อ 13 คุ้นๆนะครับ อ้อ สสวท. ปี '38 รึเปล่า ประมาณปีแถวๆนั้นแหละครับ
จัดรูป $\large \frac{1}{k^2+k+1}=\frac{(k+1)-k}{1+k(k+1)}$
เห็นแล้วคงนึกถึงสูตรของ tan ใช่แล้วครับ จัดรูปหน่อยก็จะได้คำตอบ :D

Mastermander 25 เมษายน 2006 23:50

$$\because \arctan x - \arctan y = \arctan\frac{x-y}{1+xy}$$
$$\therefore \arctan\frac{k+1-k}{1-k(k+1)}=\arctan k+1 - \arctan k$$
$$\sum_{k=1}^{2546} \arctan (k+1) - \arctan k=\arctan2547-\arctan 1$$
$$\arctan x -\frac{\pi}{4} = \arctan2547-\arctan 1$$
$$x=2547$$

14.จงแก้อสมการ
$$\sin x < \cos x$$

R-Tummykung de Lamar 26 เมษายน 2006 01:09

ความจริงข้อนี้ดูจากกราฟแล้วก็ตอบได้เลย แต่ผมขอคิดแบบ algebric ดีกว่า (เรียกแบบนี้รึเปล่า อิอิ)

คูณตลอดด้วย $\large \sin \frac{\pi}4=\cos \frac{\pi}4>0$

$$\sin x \cos \frac{\pi}4- \cos x \sin\frac{\pi}4<0$$
$$\sin (x-\frac{\pi}4)<0$$
นั่นคือ $\large x-\frac{\pi}4 \in(-\pi+2n\pi,2n\pi)\quad,n\in I$
นั่นคือ $\large x \in( (2n-\frac 34)\pi,(2n+\frac 14)\pi)\quad,n\in I$

หุหุ ไหนๆทำข้อนี้แล้ว เอากราฟมาให้ดูดีกว่า

หมายเหตุ สีเขียวเป็นกราฟของ $\cos$ สีฟ้าเป็นกราฟของ $\sin$
จุดที่ $\large \sin x=\cos x$ คือ $\frac{\pi}4+n\pi$
ก็จะได้คำตอบเดียวกันครับ คือ
$\large x \in( (2n-\frac 34)\pi,(2n+\frac 14)\pi)\quad,n\in I$

R-Tummykung de Lamar 27 เมษายน 2006 01:58

ขอใช้สิทธิ์ตั้งคำถามข้อต่อไปละกันครับ

สามเหลี่ยมที่มีด้านทั้งสามยาว $$\large \sin \theta,\sin 2\theta,\sin 3\theta\qquad,0\leq \theta <2\pi$$
$(i)\ \ $ มีพื้นที่เท่าไหร่ครับ
$(ii)\ \ $ มีพื้นที่มากที่สุดเท่าไหร่ครับ

(from mathlinks)

warut 29 เมษายน 2006 12:46

ข้อนี้ผมใช้คอมพิวเตอร์ช่วยคำนวณได้มันหยดเลยครับ ได้ผลดังนี้

1. พื้นที่ของสามเหลี่ยมคือ $$\frac{\sin \theta \sin 2\theta \sin 3\theta}{2}$$
2. พื้นที่มากที่สุดคือ $$\frac{34\sqrt2+ 5\sqrt5}{216}$$
ไม่เข้าใจเหมือนกันครับว่า ทำไมคำตอบของข้อ 1. มันออกมาได้พอดิบพอดีอย่างงั้น :rolleyes:

R-Tummykung de Lamar 30 เมษายน 2006 18:07

ข้อ 1 ถูกต้องแล้วครับ ส่วนข้อ 2 ยังไม่ทราบเหมือนกันครับ
และข้อ 1 ถ้าอยากดูแนวคิดก็ ที่นี่เลยครับ
Area of triangle

nithi_rung 01 พฤษภาคม 2006 01:21

ได้คำตอบ $ \frac{34\sqrt2+ 5\sqrt5}{216} $ เท่ากับพี่ warut ครับ

ให้ $ x=\sin\theta $ และพิจารณา
$ g(x)=\frac{1}{2}\sin\theta\sin 2\theta\sin 3\theta=x^3(3-4x^2)\sqrt{1-x^2} $
เมื่อ $ x\in\left(0,\frac{\sqrt3}{2}\right) $
หาอนุพันธ์ของ $g(x)$ จะได้ว่าค่าสูงสุดอยู่ที่ $ x^2=\frac{8-\sqrt{10}}{12} $
จะได้พื้นที่สามเหลี่ยมมากที่สุดคือ $ \frac{34\sqrt2+ 5\sqrt5}{216} $ ตารางหน่วย โดยเกิดสมการเมื่อ $ \sin^2\theta=\frac{8-\sqrt{10}}{12} $

แหะๆ วิธีไม่ค่อยสวยงามเท่าไหร่ มีใครมีวิธีอื่นอีกรึเปล่าครับ

warut 01 พฤษภาคม 2006 07:15

ผมก็ทำคล้ายๆกันนี่แหละครับ แต่ผมหาจุดสูงสุดของ $\{g(x)\}^2$ แทน $g(x)$ ซึ่งอาจช่วยให้ง่ายขึ้นเล็กน้อยครับ (ไม่ต้องติด square root)


เวลาที่แสดงทั้งหมด เป็นเวลาที่ประเทศไทย (GMT +7) ขณะนี้เป็นเวลา 01:49

Powered by vBulletin® Copyright ©2000 - 2024, Jelsoft Enterprises Ltd.
Modified by Jetsada Karnpracha